Government Accounting Missed Questions Flashcards
A city government reported a $9,000 increase in net position in the motor pool internal service fund, a $12,000 increase in net position in the water enterprise fund, and a $7,000 increase in the employee pension fund. The motor pool internal service fund provides service primarily to the police department. What amount should the city report as the change in net position for business-type activities in its statement of activities?
A. $9,000 B. $12,000 C. $21,000 D. $28,000
The correct answer is (B).
Business type activities that are normally financed through user charges and reports the consolidated results of all enterprise funds. It does not include internal service funds, which are accounted for in governmental activities. The employee pension fund is a fiduciary fund and not included in business-type activities. The $12,000 increase in net position from Water enterprise is included.
(A) is incorrect because the motor pool internal service fund is excluded from business-type activities.
(C) is incorrect because the calculation also includes motor pool internal service fund which should be excluded.
(D) is incorrect because the motor pool Internal service fund and employee pension trust fund should be excluded from business-type activities.
Oak County incurred the following expenditures in issuing long-term bonds:
Issue cost $ 400,000
Debt insurance 90,000
When Oak establishes the accounting for operating debt service, what amount should be deferred and amortized over the life of the bonds?
A.$90,000
B.$0
C.$400,000
D.$490,000
The correct answer is (B).
The general long-term debt of a state or local government is reported:
Government-wide financial statements: Accrual basis of accounting.
Governmental Funds (Debt Service Funds): Modified Accrual basis of accounting.
In Government Funds (Debt Service Funds) under Modified Accrual basis of accounting costs are recognized using the expenditure principle, and recorded in the period the obligation to pay arises, irrespective of when the goods/services are being actually used by the government entity.
Accordingly, the modified accrual basis does not allow for amortization of issue costs and debt insurance premiums, which must be recognized in full when the liability is incurred.
The amount deferred and amortized over the life of the bonds is $0.
$490,000 is expensed in the period incurred.
Users need information on how the government applies restricted resources when they have a choice between using restricted or unrestricted resources to evaluate which of the following?
A.Whether the government made improper payments from the restricted and unrestricted resources
B.Whether the government complied with laws and regulations associated with federal funds
C.How ending balances may be applied to continuation of programs and services in future periods
D.None of the above
The correct answer is: C
Explanation:
Governments rely on grants and other types of contributions to fund a variety of programs. The note disclosing how the government applies restricted resources when they have a choice between using restricted versus unrestricted resources provides users with information to evaluate how ending balances may be applied to continuation of programs and services in future periods (not whether the government made improper payments or whether the government complied with laws and regulations).
On January 1, Fonk City approved the following general fund resources for the new fiscal period:
Property taxes $ 5,000,000 Licenses and permits 400,000 Intergovernmental revenues 150,000 Transfers in from other funds 350,000 What amount should Fonk record as estimated revenues for the new fiscal year?
A.$5,400,000
B.$5,550,000
C.$5,750,000
D.$5,900,000
The correct answer is: B
Explanation:
The correct answer is B. Estimated revenues are revenues that are expected to be available to be spent in the period.
Ref Summary Amount
a Property taxes $5,000,000
b Licenses & Permits $400,000
c Inter-governmental revenues $150,000
d Estimated Revenue (a+b+c) $5,550,000
Transfers in from other funds are not considered because it is an estimated other financing source, not revenue. Inter-governmental revenue is included in the calculation of estimated revenue. Transfers in from other funds of $350,000 are not included in the calculation for estimated revenue.
Which of the following is included in the governmental fund statements?
A.Cash flows statements
B.Component unit information
C.Reconciliation between the fund statements and the government-wide statements
D.Major fiduciary funds
The correct answer is: C
Explanation:
Governmental fund financial statements present a reconciliation to the government-wide statements. Cash flows statements appear only in the fund statements for proprietary funds. Individual component unit information is presented in individual columns in the government-wide statements, after the primary government’s fund statements, or in the notes to the financial statements. Governmental fund financial statements do not include major fiduciary funds. Even fiduciary fund statements present fund types, not major funds.
One feature of state and local government accounting and financial reporting is that fixed assets used for general government activities
A.
Often are not expected to contribute to the generation of revenues.
B.
Do not depreciate as a result of such use.
C.
Are acquired only when direct contribution to revenues is expected.
D.
Should not be maintained at the same level as those of businesses so that current financial resources can be used for other government services.
The correct answer is: A
Explanation:
Governmental entities invest large amounts of resources in non-revenue producing capital assets such as government office buildings, highways, bridges, and sidewalks.
A government’s assets include inventory of $2 million, roads constructed for $25 million with accumulated depreciation of $10 million, and equipment acquired for $5 million with accumulated depreciation of $1 million. Its liabilities include an outstanding balance of $5 million for bonds payable issued to construct the roads and a $1 million short-term loan for inventory purchases. What amount should be reported as the net investment in capital assets in the government-wide statement of net position?
A.$26 million
B.$25 million
C.$14 million
D.$10 million
The correct answer is: C
Explanation:
The correct answer is (C).
$14,000,000 should be reported as the net investment in capital assets in the government-wide statement of net position. Net assets on the government-wide statement of financial position are divided into - Restricted, Unrestricted and Net Investment in Capital Assets.
Assets:
Roads Constructed $25,000,000
Accum. Depreciation ($10,000,000) $15,000,000
Equipment $5,000,000
Accum. Depreciation ($1,000,000) $4,000,000
Liabilities:
Bonds Payable $(5,000,000)
*Net Investment in Capital Assets $14,000,000
Note: Inventory is not a capital asset and therefore both the inventory and liability related to the purchase of inventory is also excluded.
Which basis of accounting is required for a city’s government-wide financial statements?
A. Cash. B. Modified Cash. C. Modified Accrual. D. Accrual.
The correct answer is: D
Explanation:
A city’s government-wide financial statements are required to be completed on the accrual basis of accounting.
Except in specified circumstances, interest costs on long-term liabilities are reported under governmental accounting rules in which of the following manners?
A. Separately as a direct program cost B. Separately as an indirect cost C. In a particular note to the financial statements D. None of the above
The correct answer is: B
Explanation:
In general, interest costs on long-term liabilities will be reported separately as an indirect cost (not in a particular note). Note disclosures will include information about how interest costs are reported and what amounts are included as direct expenses.
GASB Statement No. 34, Basic Financial Statements and Management’s Discussion and Analysis for State and Local Governments, requires large governments (annual revenues in the range above $100 million) to
Report all fixed assets (land, buildings, and equipment).
In fiscal periods beginning after June 15, 2001, report major general infrastructure asset acquisitions.
In fiscal periods beginning after June 15, 2005, retroactively report major general infrastructure assets acquired in fiscal years ending after June 30, 1980, and before adoption of GASB Statement No. 34.
A.
I only
B.
I and II only
C.
I and III only
D.
I, II, and III
The correct answer is: D
A city received a $9,000,000 federal grant to finance the construction of a homeless shelter. In which fund should the proceeds be recorded?
A. Permanent B. General C. Capital projects D. Special revenue
Explanation:
The correct answer is (C).
The $900,000 the city received as a federal grant to finance the construction of a homeless shelter should be recorded in the capital projects fund. Capital projects fund accounts for major acquisition or construction activities of capital assets. A capital projects fund is used to establish resources to construct or acquire major, long-lived general government facilities. The construction of a homeless shelter via a federal grant is a perfect example.
Which of the following factors influences governmental generally accepted accounting and reporting principles?
A.The lack of SEC oversight for municipal financial instruments
B.State statutes that created the Governmental Accounting Standards Board (GASB)
C.Governmental structure and its mission to provide critical public services
D.Population levels for individual governments
The correct answer is: C
Explanation:
The essential mission of government is delivering services, financed with taxes. The SEC does have limited oversight for tax-exempt financial instruments, but the GASB is focused on the operating and organizational characteristics to define governmental accounting and reporting standards. The GASB was created by the Financial Accounting Foundation and is not subject to any individual state regulation. While the GASB has sometimes considered “small GAAP” versus “big GAAP,” the size of individual state and local governments is not a key factor for the Conceptual Framework.
Which fund would report the amount of bonds payable that are due within one year?
A. Governmental funds B. Proprietary funds C. Both governmental and proprietary funds D. Neither governmental nor proprietary funds
The correct answer is: B
Explanation:
Proprietary funds report the current amounts due for any bonds outstanding on its statement of net position. At the fund level, governmental funds do not report the current portions of principal or interest due.
Several disclosures in the Significant Accounting Policies section of governmental financial statements describe which of the following?
A.
How the government complied with applicable laws and regulations
B.
Why the government exceeded its statutory budgetary authority
C.
Accounting policies for recognizing revenues and expenditures
D.
Why the notes are not necessary for a complete understanding of the government’s financial statements
The correct answer is: C
Explanation:
The Significant Accounting Policies section includes a number of explanations about what policies the government has adopted for recognition, such as how revenues are recognized, asset capitalization, program revenues, and the government’s definition of operating transactions. Disclosures do not describe how the government complied with applicable laws and regulations, why the government exceeded its statutory budgetary authority, or why the notes are necessary.
Which of the following indicates why fiduciary funds are presented in the fund statements instead of the note disclosures?
A. To enhance municipal bond disclosures B. To improve budgetary compliance C. To provide accountability for these resources D. To comply with federal grant reporting requirements
The correct answer is: C
Explanation:
The GASB considered using note disclosures to present fiduciary funds, but decided that financial statement presentations would enhance stewardship and accountability for these resources. Municipal bonds are not secured with fiduciary fund resources since these assets are not available for the government’s programs. Fiduciary funds are generally not required to adopt budgets. Federal grants are not generally classified as fiduciary funds.
Central County received proceeds from various towns and cities for capital projects financed by Central’s long-term debt. A special tax was assessed by each local government, and a portion of the tax was restricted to repay the long-term debt of Central’s capital projects. Central should account for the restricted portion of the special tax in which of the following funds?
A. Internal service fund B. Enterprise fund C. Capital projects fund D. Debt service fund
Explanation:
The correct answer is (D).
Debt service funds account for the accumulation of resources for, and the payment of, general long-term debt principal and interest.
Therefore, the portion of the special tax that is restricted to repay the long-term debt of the government’s capital projects fund should be accounted for in a debt service fund.
Internal service funds are used to account for the activities of a department that provides services to other departments of the same government in a manner similar to a business. Internal service funds (a type of proprietary fund) account for only their own assets, liabilities, revenues, and expenses.
Enterprise funds are used to account for the activities of a department that provides services to others in a manner similar to a business. Enterprise funds (a type of proprietary fund) account for only their own assets, liabilities, revenues, and expenses.
Capital project funds are used to account for the construction of capital projects.
Which event(s) is(are) supportive of interperiod equity as a financial reporting objective of a governmental unit?
A balanced budget is adopted. Residual equity transfers out equals residual transfers in. A. I only B. II only C. Both I and II D. Neither I nor II
The correct answer is: A
Explanation:
Interperiod equity is the idea of using the revenues from one period to pay for the expenditures of that same period and that period only. A balanced budget is a plan for this occurrence. Residual equity transfers are between funds, not between periods.
Which of the following characteristics describe the financial resources for capital projects funds?
A.
Revenues based on rates that will recover the cost of capital assets required to deliver services
B.
Bond issue proceeds, grants, or interest earnings
C.
Donations restricted, such that only investment earnings can be used for maintenance
D.
Prior year encumbrances
The correct answer is: B
Explanation:
Resources for capital project funds may arise from bond issue proceeds, grants, or interest earnings. Revenues based on rates that will recover the costs of capital assets required to deliver services describes the approach used in enterprise fund accounting. Donations restricted such that only investment earnings can be used for maintenance would be accounted for in a permanent fund. The encumbrance system is used in governmental funds (general, special revenue, and capital projects funds) to prevent over-expenditure and to demonstrate compliance with legal requirements. Encumbrances are a budgetary measurement element, not a criterion for determining fund type.
Which of the following should be included in financial reports to help users assess the government’s operating results?
A. Cash flow statements B. Other supplementary information C. Management discussion and analysis D. Explanation of the sources and uses of financial resources
The correct answer is: D
Explanation:
The second reporting objective discussed in Concepts Statement No. 1 suggests that governmental financial reporting should assist users in evaluating the government’s operating results by explaining sources and uses of financial resources. Cash flow statements are a specific element in proprietary fund statements and are not discussed in Concepts Statement No. 1. Other supplementary information is useful for enhancing financial statements, but is not necessarily required. The management discussion and analysis is one of the components of required supplementary information, but is not discussed in Concepts Statement No. 1.
Which of the following would be reported using the full accrual method of accounting?
A. Governmental activities B. Business-type activities C. Neither governmental activities nor business-type activities D. Both governmental activities and business-type activities
The correct answer is: D
Explanation:
Both governmental activities and business-type activities are included on the face of the government-wide financial statements. The government-wide financial statements use the full accrual method of accounting, which means that D is the correct answer. All amounts on the government-wide financial statements are reported using the accrual method.
Lake County received the following proceeds that are legally restricted to expenditure for specified purposes:
Levies on affected property owners to install sidewalks $500,000
Gasoline taxes to finance road repairs 900,000
What amount should be accounted for in Lake’s special revenue funds?
A. $1,400,000 B. $900,000 C. $500,000 D. $0
The correct answer is: B
Explanation:
Special revenue fund sources includes fees, grants, specific taxes, and other earmarked revenue sources. The gasoline taxes to finance road repairs for $900,000 would be a specific tax, accounted under special revenue funds. Levies on affected property owners to install sidewalks is usually accounted for under the agency fund. It may also be accounted for in a debt service fund, for example: the levies are used to repay a bond that was issued to install the sidewalks. Options (a), (c) and (d) are incorrect based on the above explanation.
All of the following statements regarding notes to the basic financial statements of governmental entities are true except
A.
The notes contain disclosures related to required supplementary information.
B.
Some notes presented by governments are identical to notes presented in business financial statements.
C.
Notes that are considered essential to the basic financial statements need to be presented.
D.
It is acceptable to present notes in a very extensive format.
The correct answer is: A
Explanation:
The notes to the basic financial statements of governmental entities do not contain disclosures related to required supplementary information. The Management’s Discussion and Analysis (MD&A) is the required supplementary information in the government’s general purpose external report and it is presented before the financial statements. .
Fixed assets of an enterprise fund should be accounted for in the
A.
Enterprise fund but no depreciation on the fixed assets should be recorded.
B.
Enterprise fund and depreciation on the fixed assets should be recorded.
C.
General fixed asset account group and depreciation on the fixed assets should be recorded.
D.
General fixed asset account group but no depreciation on the fixed assets should be recorded.
The correct answer is: B
Explanation:
The enterprise fund is a self-supporting fund which provides goods and/or services to the general public. Revenues and expenses are recorded in the same manner as in commercial business enterprises. Fixed assets are recorded in the fund as well as the associated depreciation charges.
Maple City’s public employee retirement system (PERS) reported the following account balances at June 30:
Reserve for employer’s contributions $5,000,000
Actuarial deficiency in reserve for employer’s contributions 300,000
Reserve for employees’ contributions 9,000,000
Maple’s PERS fund balance at June 30, should be
A.
$ 5,000,000
B.
$ 5,300,000
C.
$14,000,000
D.
$14,300,000
The correct answer is: D
Explanation:
The PERS should be accounted for in a pension trust fund. Under the traditional approach of accounting for pension trust funds under NCGA Statement 1, the $14,300,000 fund balance of the pension trust fund can be determined by adding the reserve for employee contributions (i.e., $9,000,000), the reserve for employer contributions (i.e., $5,000,000), and the actuarial deficiency in reserve for employer’s contributions (i.e., $300,000).
Gold County received goods that had been approved for purchase but for which payment had not yet been made. Should the accounts listed below be increased?
Encumbrances Expenditures A No No B No Yes C Yes No D Yes Yes
The correct answer is: B
Explanation:
The following entries would be made by a governmental unit when ordering and receiving goods:
To record purchase order issued when goods were approved for purchase:
Encumbrances (expected cost) XX
Reserve for encumbrances XX
To record expenditure upon receipt of the goods:
Reserve for encumbrances XX
Encumbrances XX
Expenditures (actual) XX
Vouchers payable XX
Thus, upon receipt of the goods, the Encumbrance account decreases and the Expenditures account increases.
What is the measurement focus and the basis of accounting for the government-wide financial statements?
Measurement focus Basis of accounting A Current financial resources Modified accrual B Economic resources Modified accrual C Current financial resources Accrual D Economic resources Accrual
The correct answer is: D
Explanation:
Government-wide financial statements aggregate information for all governmental and business-type activities. GASB 34 requires an economic resources measurement focus and accrual basis of accounting for all amounts in the government-wide financial statements.
Which of the following indicates when the fund balance may not be reported as assigned?
A. If the assignment causes a deficit in unassigned fund balance B. In Capital Projects funds C. If there is no component for committed fund balance D. In the General Fund
The correct answer is: A
Explanation:
The correct answer is (A)
Assigned fund balance is based on management’s intent, but cannot create a deficit in unassigned fund balance. Assigned fund balance cannot set aside resources that are not available. Assigned fund balance may be reported in any governmental fund. Residual balances in special revenue funds may be classified as assigned since management has created these funds for specific purposes. There is no requirement for a committed fund balance before assignments may be reported.
Kingwood Town paid $22,000 cash for a flatbed trailer to be used in the general operations of the town. The expected useful life of the trailer is 6 years with an estimated $7,000 salvage value. Which of the following amounts would be reported?
A.
$15,000 increase in equipment in the general fund.
B.
$15,000 increase in investment in governmental activities column for fixed assets.
C.
$22,000 increase in investment in governmental activities column for fixed assets.
D.
$22,000 increase in equipment in the general fund.
The correct answer is: C
Explanation:
Purchased assets are reported in the governmental activities column at cost ($22,000). Long-term (capital) assets are not recorded in any governmental fund.
Which of the following characteristics describe a multiple plan management of a PERS?
A.
Assets accumulated for benefit payments are available to pay benefits for any of the plan members.
B.
Assets are accumulated in separate reserves restricted for benefits of specific classes or groups of employees.
C.
The government serves as a custodian for the accumulated assets and distributes funds based on instructions from member governments.
D.
Assets are not accessible to creditors of member governments.
The correct answer is: B
Explanation:
The PERS is responsible for multiple plans when assets are accumulated in separate reserves restricted for benefits of specific classes or groups of employees. Assets being accumulated for benefit payments available to pay benefits for any of the plan members is a characteristic of the single plan. The government serving as a custodian for the accumulated assets and distributing funds based on instructions from member governments describes an agency fund. Creditor access is not a distinguishing feature for pension trust plans.
Which of the following statements is the most significant characteristic in determining the classification of an enterprise fund?
A.
The predominant customer is the primary government.
B.
The pricing policies of the activity establish fees and charges designed to recover its cost.
C.
The activity is financed by debt that is secured partially by a pledge of the net revenues from fees and charges of the activity.
D.
Laws or regulations require that the activity’s costs of providing services including capital costs be recovered with taxes or similar revenues.
The correct answer is: B
Explanation:
Enterprise funds must be used to account for a government’s business-type operations that are financed and operated like private businesses—where the government’s intent is that all costs (expenses, including depreciation) of providing goods or services to the general public on a continuing basis are to be financed or recovered primarily through user charges (operating revenue). Most government-owned public utilities (e.g., electricity, gas, water, and sewage systems) must be accounted for as enterprise funds.
Which of the following indicates when the governmental fund balance may be reported as committed?
A.
Constraints are imposed by external parties through grants or contracts
B.
Constraints are imposed by formal action of the government’s governing body
C.
Amounts are set aside for specific purposes based on management’s intent
D.
Amounts have not been restricted or assigned
The correct answer is: B
Explanation:
Committed fund balances reflect amounts that can only be used for specific purposes based on constraints imposed by formal action of the government’s highest level of decision-making authority. Constraints imposed by external parties through grants or contracts are reported as restricted fund balance, not committed. Amounts set aside for specific purposes based on management’s intent are reported as assigned fund balance, not committed. A restriction or assignment is not a prerequisite for fund balance classifications.
A county’s balances in the general fund included the following:
Appropriations $435,000 Encumbrances 18,000 Expenditures 164,000 Vouchers payable 23,000 What is the remaining amount available for use by the county?
A. $230,000 B. $248,000 C. $253,000 D. $271,000
The correct answer is: C
Explanation:
Funds available for use = Appropriations (budgeted) - Encumbrances - Expenditures ( $435,000 - $18,000 - $164,000 ) = $253,000. Option (a) is incorrect because vouchers payable is excluded in available for funds calculations. Option (b) is incorrect because encumbrances are not excluded, instead vouchers payable are excluded from appropriations. Option (d) is incorrect because encumbrances should be excluded from appropriations to arrive at funds available.
During the current year, Vann County’s motor pool internal service fund sold two vehicles for $5,000. The vehicles had a cost of $6,000 and a carrying value of $4,000. How should Vann County’s motor pool internal service fund report this transaction in its fund financial statements?
A. Revenue of $5,000 B. Other financing source of $5,000 C. Special item of $1,000 D. Gain of $1,000
The correct answer is: D
Explanation:
The correct answer is (D).
The motor pool internal service fund is reported under the proprietary fund in the fund financial statements. The proprietary fund uses the accrual basis of accounting which records operating revenues, operating expenses and non-operating revenues and non-operating expenses to get the change in net position. Vann County’s motor pool internal service sold two vehicles for $5,000. Both vehicles together have a cost of $6,000 and a carrying value of $4,000. Because the carrying value was $4,000 and the vehicles sold for $5,000, Vann County’s motor pool internal service fund should report a gain of $1,000.
A special-purpose government is distinguished from a general purpose government by which of the following?
A. It is established by a state government. B. It provides limited services, such as only governmental or business-type activities. C. It does not manage any fiduciary funds. D. It is not governed by a separately elected legislative body.
The correct answer is: B
Explanation:
The distinction between general purpose and special-purpose governments is based on the entity’s programs and services, not how the government was created. Special-purpose governments only provide limited services; unlike cities, counties, states, towns or villages that provide a broad range of services. It is possible that a special purpose government may only manage fiduciary programs. Governance structure is not the distinguishing feature of special-purpose governments.
In the General Fund Statement of Revenues, Expenditures, and Changes in Fund Balances, which of the following has an effect on the excess of revenues over expenditures?
A. Purchase of fixed assets B. Payment to a debt-service fund C. Special items D. Proceeds from the sale of capital assets
Explanation:
The correct answer is (A).
The General Fund uses modified accrual accounting, in which the purchase of a fixed asset is an expenditure, which would affect the excess of revenue over expenditure.
Option (B) is incorrect because transfers out are reported as other financing uses which are presented below the excess of revenues (deficiency) over expenditure.
Option (C) is incorrect because special items are presented below the excess of revenues (deficiency) over expenditure.
Option (D) is incorrect because proceeds from the sale are reported in special items, which is presented below the excess of revenues (deficiency) over expenditure.
Which of the following statements about the statistical section of the Comprehensive Annual Financial Report (CAFR) of a governmental unit is true?
A.
Statistical tables may not cover more than two fiscal years.
B.
Statistical tables may not include nonaccounting information.
C.
The statistical section is not part of the basic financial statements.
D.
The statistical section is an integral part of the basic financial statements.
The correct answer is: C
Explanation:
The basic financial statements include both the government-wide statements and the fund statements, as well as the notes, but not the statistical section.
Which of the following is considered part of the required supplementary information for general-purpose external financial reporting of a local government?
A. Fund financial statement. B. Combining non-major fund statement. C. Notes to the financial statements. D. Management discussion and analysis.
Explanation:
The correct answer is (D).
Required Supplementary Information is the information that is mandated to be disclosed by the reporting entity. This information, though not a part of the financial statements, are considered a prerequisite for a complete understanding and overview of financial information.
One such information is Management Discussion and Analysis (MD&A), which is the assessment of the organization’s performance for the said period by the management. MD&A are a compulsory part of Required Supplementary Information for general purpose external financial reporting of a local government.
Smith City levied property taxes totaling $1,000,000 during the current year, of which $150,000 was collected within 60 days after year-end. What amount should Smith City record as property tax revenue?
A. $850,000 B. $1,000,000 C. $150,000 D. $0
The correct answer is: B
Explanation:
The correct answer is (B).
Property taxes are considered the enforceable legal claim and is always recognized when billed. A government can expect to collect the amount billed to taxpayers once a property tax assessment is made as these taxes are mandatory.
As a result, property taxes that have been billed usually satisfy the requirements of being measurable and available, and are accrued in advance of collection. However, an allowance for uncollectible must be established.
Dr. Property Taxes Receivable-Current $1,000,000
Cr. Property Tax Revenue $1,000,000
Which of the following items is an example of imposed nonexchange revenue for a governmental entity?
A. Personal income taxes B. Retail sales tax C. Federal grant money D. Property taxes
The correct answer is: D
Explanation:
Imposed nonexchange revenues represent assessments imposed on non-governmental entities and include property taxes and fines or forfeitures. Taxes on earnings or consumption are classified as derived tax revenues. Certain grants, entitlements, and donations are classified as voluntary nonexchange transactions. Voluntary nonexchange transactions result from legislative or contractual agreements involving parties that may or may not be governmental entities, but do not involve an exchange of equal value.
If a city government is the primary reporting entity, which of the following is an acceptable method to present component units in its combined financial
statements?
A. Consolidation. B. Cost method. C. Discrete presentation. D. Government-wide presentation.
The correct answer is: C
Explanation:
Government-wide statements (GWS) aggregate information for all governmental and business-type activities. GASB 34 requires an economic resources
measurement focus and accrual basis of accounting for all amounts in the GWS. There are four required columns in the GWS, one each for: governmental
activities, business-type activities, the primary government (sum of the previous two), and component units. Most component units should be included in the
financial reporting entity by discrete presentation (reported in columns separate from primary government).
The debt service fund of a governmental unit is used to account for the accumulation of resources for, and the payment of, principal and interest in connection with a
Private-purpose trust fund Proprietary funds A No No B No Yes C Yes Yes D Yes No
The correct answer is: A
Explanation:
Only general obligation long-term debt should be serviced through debt service funds. Fiduciary and proprietary fund debt are rarely general government obligations.
The mission of the Governmental Accounting Standards Board (GASB) includes which of the following?
A.
Providing a framework to educate users and support public policies, invest-ments, and management by those users
B.
Establishing standards to identify appropriate performance measures and criteria
C.
Outlining best practices for administration of governmental budgets
D.
Providing accounting and reporting standards for federal government agencies
The correct answer is: A
Explanation:
The GASB’s mission includes research and development of standards that will improve accounting and financial reporting for state and local governments. The GASB’s mission includes providing a framework to educate users and support public policies, investments, and management by those users.
Fund accounting is used by governmental units with resources that must be
A.
Composed of cash or cash equivalents.
B.
Incorporated into combined or combining financial statements.
C.
Segregated for the purpose of carrying on specific activities or attaining certain objectives.
D.
Segregated physically according to various objectives.
The correct answer is: C
Explanation:
Fund accounting is used when there are legal separations between sources and uses of funds. Fund accounting may be used to account for assets aside from cash and cash equivalents. Combined or combining financial statements may be used with or without fund accounting. The resources may be physically in the same account or location.
Which of the following is the paramount objective of financial reporting by state and local governments?
A. Reliability B. Consistency C. Comparability D. Accountability
The correct answer is: D
Explanation:
Governments are focused on programs and services that will improve the quality of life in their communities. Governmental entities must provide accountability for how they raised resources for those programs and explain how they managed and safeguarded those public resources.
Zephyr City has adopted GASB Statement No. 34, Basic Financial Statements and Management’s Discussion and Analysis for State and Local Governments. Which activities and basis of accounting must appear in Zephyr’s statement(s) of cash flows?
Business-type activities on the accrual basis in the fund financial statements
Government-type activities on the modified accrual basis in the fund financial statements
Business-type and government-type activities on the accrual basis in the government-wide financial statements
A.
I only
B.
II only
C.
III only
D.
I and III only
The correct answer is: A
Explanation:
Governmental-type funds pose problems for developing a meaningful government-wide cash flows statement. Only funds for business-type activities are required in the statement of cash flows.
Which of the following is a significant issue for SEA reporting for governmental entities?
A.
SEA reporting provides authorization for tax initiatives.
B.
SEA reporting will help explain budgetary proposals.
C.
SEA reporting helps bring performance results information to citizens to help with their assessment, program selection, and implementation.
D.
SEA reporting is required by GASB accounting and reporting standards.
The correct answer is: C
Explanation:
SEA reporting emphasizes performance results and is designed to help public policy-makers evaluate which programs are critical for a community.
Park City uses encumbrance accounting and formally integrates its budget into the general fund’s accounting records. Park incurred salaries and wages of $800,000 for the month of April. What account should Park debit to record this $800,000?
A. Encumbrances control B. Salaries and wages expense control C. Expenditures control D. Operating funds control
The correct answer is: C
Explanation:
While goods and services committed for by purchase order or contract are encumbered in governmental funds to avoid overspending appropriations, some expenditures are controlled by other means and need not be encumbered. Salaries and wages are set by contract and controlled by established payroll procedures and are not encumbered. Salaries and wages expense would not be appropriate for a governmental-type fund. Salaries and wages should be recorded as follows:
Expenditures control 800,000
Vouchers payable 800,000
Governmental budgetary comparisons may be presented as required supplementary information (RSI) or which of the following?
A. As a reconciling item in the statement of net assets B. As supplementary information C. In the notes to the financial statements D. In the basic financial statements
The correct answer is: D
Explanation:
Governments are encouraged to present budgetary comparisons as part of required supplementary information (RSI). There was significant discussion about whether these reports should be part of the basic financial statements, in part to ensure that audit coverage provided additional credibility for the reports. The GASB allowed governments to elect to present budgetary comparisons as statements within the basic financial statements, but not as part of other supplementary information or notes to the financial statements.
The following information pertains to Pine City’s general fund for the current year:
Appropriations $6,500,000 Expenditures 5,000,000 Other financing sources 1,500,000 Other financing uses 2,000,000 Revenues 8,000,000 After Pine's general fund accounts were closed at the end of the year, the unassigned fund balance increased by
A. $3,000,000 B. $2,500,000 C. $1,500,000 D. $1,000,000
The correct answer is: B
Explanation:
Appropriations is a budgetary account which will also be closed. However, since closing the budgetary accounts (which are not all given) simply reverses the
entry to record the budget, their closing has no effect on fund balance. The closing entry for the activity accounts given would increase the fund balance, as
follows:
Revenues 8,000,000 Other financing sources 1,500,000 Expenditures 5,000,000 Other financing uses 2,000,000 Unassigned fund balance (difference) 2,500,000
Which of the following is not reported in the enterprise fund Statement of Net Position?
A. Net investment in capital assets B. Restricted C. Unrestricted D. Capital contributions
The correct answer is: D
Explanation:
Capital contributions is not a category used on the government-wide statement of net position. Net position will be displayed in the same three categories that we used on the government-wide statement of net position: invested in capital assets, net of related debt; restricted net position; and unrestricted net position.
Elm City contributes to and administers a single-employer defined benefit pension plan on behalf of its covered employees. The plan is accounted for in a pension trust fund. Actuarially determined employer contribution requirements and contributions actually made for the past three years, along with the
percentage of annual covered payroll, were as follows:
Contribution made Actuarial requirement
Amount Percent Amount Percent
Year 3 $11,000 26 $11,000 26
Year 2 5,000 12 10,000 24
Year 1 None None 8,000 20
What account should be credited in the pension trust fund to record the year 3 employer contribution of $11,000?
A. Revenues control B. Other financing sources control C. Due from special revenue fund D. Pension benefit obligation
The correct answer is: A
Explanation:
All pension trust fund contributions and earnings are accounted for as fund revenues. The pension trust fund makes the following entry to record employer contributions:
Cash 11,000
Revenues control—employer contribution 11,000
A local government has the following liabilities on its adjusted trial balance at the end of the current year. What amount would the government report on its balance sheet as total liabilities? $50,000 accrued payroll $100,000 bonds payable – current portion $900,000 bonds payable – long-term portion $25,000 accounts payable A. $1,075,000 B. $175,000 C. $100,000 D. $75,000
The correct answer is: D
Explanation:
The balance sheet is a fund level financial statement and thus uses the modified accrual method of accounting. This means that the $1,000,000 related to the bonds will not be reported, and the answer is the $50,000 in accrued payroll and the $25,000 in accounts payable.
The water department of a city provides water to the nearby districts for a monthly fee. In year 2, it decides to take machinery on lease from one of the firms in the city. The lease term was decided to be 3 years and the ownership of the machinery would not transfer to the department at the end of the term. Given that the useful life of the machinery was 2 years and the value of the leased asset was $120,000 in the books of the water department, what would be the amortization in the books in year 2?
A. $60,000 B. $40,000 C. $120,000 D. $0
The correct answer is: A
Explanation:
The correct answer is (A).
In leases other than a short-term lease and contracts that transfer ownership, the lessee recognizes a right-to-use asset in its books. The asset is required to be amortized over the life of the asset or the lease term, whichever is less. In the given question, the useful life of 2 years is less than the lease term of 3 years. The value of the ROU asset shall be amortized over a period of 2 years.
Annual amortization = 120,000/2 = $60,000
Which of the following is the purpose for governmental fund accounting?
A.
Track subsidiary profit centers
B.
Evaluate revenues and expenditures for specific programs and monitor compliance with legal restrictions
C.
Separate revenues and expenditures into as many self-supporting accounts as possible
D.
Identify statutory requirements for budget management
The correct answer is: B
Explanation:
A government’s primary mission is the delivery of programs and services. Fund accounting allows governments to track specific functions and ensure that operations comply with any legal restrictions that apply. Governments may have component units that are similar to subsidiaries in the private sector, but there are no profit objectives in governmental financial reporting. The GASB discourages the use of too many funds; the general fund is the only required fund. GASB Statement No. 34 reorganized funds to create a primary focus for each fund category, but statutory identification is not a primary objective for fund organization.
Which of the following mandates the use of an enterprise fund?
A.
Activity is financed with debt that is secured by a pledge of the net revenues from fees and charges of the activity.
B.
User fees are charged to external users for goods and services.
C.
The General Fund provides a subsidy to cover operating deficits.
D.
All surplus funds are transferred to the General Fund as a return on capital.
A.
Activity is financed with debt that is secured by a pledge of the net revenues from fees and charges of the activity.
Explanation:
GASB Statement No. 34 listed three criteria, any one of which requires the use of an enterprise fund. Activity being financed with debt that is secured by a pledge of the net revenues from fees and charges of the activity is the first of the listed criteria. User fees charged to external users for goods and services is an optional reason for using an enterprise fund. The General Fund might provide a subsidy, but this financing does not require the use of an enterprise fund. Surplus funds may be returned to the General Fund, but this transfer does not require the use of an enterprise fund.
The following information pertains to Park Township’s general fund at December 31:
Total assets, including $200,000 of cash $1,000,000 Total liabilities 600,000 Reserved for encumbrances 100,000Appropriations do not lapse at year-end. At December 31, what amount should Park report as unassigned fund balance in its general fund balance sheet? A. $200,000 B. $300,000 C. $400,000 D. $500,000
The correct answer is: B
Explanation:
Because Park’s appropriations do not lapse at year-end, the Budgetary Fund Balance Reserved for Encumbrances account is converted from an offsetting memorandum account in the general ledger to a true commitment of fund balance year-end. The amount that Park should report as Unassigned Fund Balance in its general fund balance sheet is computed as follows:
Total assets $1,000,000
Less: Total liabilities 600,000
Total fund balance $ 400,000
Less: Committed fund balance 100,000
Unassigned fund balance, December 31 $ 300,000
Which of the following is a minimum required report for the basic financial statements of a government entity?
A. Fund financial statements. B. Management's discussion and analysis. C. Required supplementary information. D. Budgetary comparison schedules.
The correct answer is: A
Explanation:
The correct answer is (A).
The various elements of the basic financial statements of a government entity are:
Government-Wide Financial Statements
Fund Financial Statements
Notes to the Financial Statements
Hence, out of the given options, fund financial statements qualifies as the minimum required report for the basic financial statements of a government entity.
Which format must an enterprise fund use to report cash flow operating activities in the statement of cash flows?
A. Indirect method, beginning with operating income. B. Indirect method, beginning with change in net position. C. Direct method. D. Either direct or indirect method.
The correct answer is: C
Explanation:
An enterprise fund is one of the governmental proprietary funds. Governments should present a statement of cash flows for proprietary funds based on the provisions of GASB statements. The GASB states the direct method of presenting cash flows from operating activities (including a reconciliation of operating cash flows to operating income) should be used.
Which of the following is one of the three standard sections of a governmental comprehensive annual financial report?
A. Investment. B. Actuarial. C. Statistical. D. Single audit.
The correct answer is: C
Explanation:
The comprehensive annual financial report contains the following sections: Introduction Section; Financial Section; and the Statistical Section.
Shared revenues received by an enterprise fund of a local government for operating purposes should be recorded as
A. Operating revenues. B. Nonoperating revenues. C. Other financing sources. D. Interfund transfers.
The correct answer is: B
Explanation:
Grants, entitlements, and shared revenues received by proprietary funds should be reported as nonoperating revenues unless they are externally restricted to capital acquisitions.
As described in GASB Concepts Statement No. 3, the distinction between required supplementary information (RSI) and other supplementary information (OSI) is generally based on which of the following?
A.
Whether the GASB has specified the requirement to present the information
B.
Whether the government has decided to present the information
C.
The public’s demand for information from the government
D.
None of the above
The correct answer is: A
Explanation:
The distinction between RSI and OSI is generally based on whether the GASB has specified the requirement to present the information, not the public’s demand for the information. Management can certainly add to RSI, but additional information must be presented separately so that users can consistently compare reports among governmental entities.
Dayne County’s general fund had the following disbursements during the year.
Payment of principal on long-term debt $100,000
Payments to vendors 500,000
Purchase of a computer 300,000
What amount should Dayne County report as expenditures in its governmental funds statement of revenues, expenditures, and changes in fund balances?
A. $300,000 B. $500,000 C. $800,000 D. $900,000
The correct answer is: D
Explanation:
Expenditures differ from expenses (as defined in commercial accounting) because expenditures include—in addition to current operating expenditures that benefit the current period—capital outlays for general fixed assets and repayment of general long-term debt principal.
Payments of principal on long-term debt $100,000
Payments to vendors 500,000
Purchase of a computer 300,000
Total expenditures $900,000
Option (a) is incorrect because payment to vendors and payment of principal on long-term debt is excluded. Option (b) is incorrect because payment of principal on long-term debt and purchase of computer is excluded. Option (c) is incorrect because payment of principal on long-term debt is excluded.
In preparing combined financial statements for a governmental entity, interfund receivables and payables should be
A.
Reported as reservations of fund balance.
B.
Reported as additions to or reductions from the unassigned fund balance.
C.
Reported as amounts due to and due from other funds.
D.
Eliminated.
The correct answer is: C
Explanation:
Interfund receivables and payables are reported as amounts due to and due from other funds. They are not reported as reservations of fund balance or additions to or reductions from the unassigned fund balance. The option of eliminating the interfund assets and liabilities is allowed, but requires that such eliminations be apparent from the headings, or be disclosed in the notes to the financial statements.
Expenditures of a governmental unit for insurance extending over more than one accounting period
A.
Must be accounted for as expenditures of the period of acquisition.
B.
Must be accounted for as expenditures of the periods subsequent to acquisition.
C.
Must be allocated between or among accounting periods.
D.
May be allocated between or among accounting periods or may be accounted for as expenditures of the period of acquisition
The correct answer is: D
Explanation:
Expenditures are recorded when fund liabilities are incurred or assets are expended, except in regard to inventory items, interest on general long-term debt, and prepaids such as insurance. This is due to the emphasis on the flow of financial resources in governmental accounting.
Which of the following transactions is an expenditure of a governmental unit’s general fund?
A.
Contribution of enterprise fund capital by the general fund
B.
Operating subsidy transfer from the general fund to an enterprise fund
C.
Routine employer contributions from the general fund to a pension trust fund
D.
Transfer from the general fund to a capital projects fund
The correct answer is: C
Explanation:
General funds use modified accrual accounting. Both capital purchases and operating expenditures are considered spending of funds and are treated as current year expenditure. Routine employer contribution to a pension trust fund is an expenditure. Option (a) is incorrect because contribution of enterprise fund capital by the general fund is not an expenditure. Option (b) is incorrect because operating transfers are reported as other financing uses under modified accrual accounting. Option (d) is incorrect because transfers out is treated as other financing uses and not expenditure
Cash receipts from grants and subsidies to decrease operating deficits should be classified in which of the following sections of the statement of cash flows for governmental, not-for-profit entities?
A. Operating B. Noncapital financing C. Capital and related financing D. Investing
The correct answer is: B
Explanation:
Noncapital financing activities include cash receipts from grants and subsidies except (1) those specifically restricted for capital purposes and (2) those for specific activities that are considered to be operating activities of the grantor government. Operating activities include cash inflows, receipts, and payments that do not result from transactions defined as capital and related financing, noncapital financing, or investing activities. Capital and related financing activities include acquiring and disposing of capital assets used in providing services or producing goods; borrowing money for acquiring, constructing, or improving capital assets and repaying the amounts borrowed, including interest; and paying for capital assets obtained from vendors on credit. Investing activities include making and collecting loans (except program loans) and acquiring and disposing of debt or equity instruments.
Which of the following characteristics trigger unique accounting standards for governmental entities?
A. Governmental ownership of utilities such as water, sewer, or electric services B. Democratic structure for state and local governmental entities C. Fund accounting and budgetary reporting D. Profit motive in governmental funds
The correct answer is: C
Explanation:
Fund accounting and budgetary reporting are key elements for demonstrating accountability for public resources and provide cornerstones for governmental financial reporting. They are the characteristics that trigger unique accounting standards for governmental entities.
Which of the following is discussed in GASB Concepts Statement No. 1 as a financial reporting objective for governmental financial statements?
A.
Assisting users in understanding funding objectives for public retirement systems and the potential tax burden on future taxpayers
B.
Assisting users in identifying management’s policies for selection of critical programs and services to provide and the annual tax burden to finance those programs
C.
Assisting users in assessing the operating results, level of services provided, and the government’s ability to meet cash requirements
D.
Assisting users in determining the annual tax burden per capita for each of the major programs provided by the governmental entity
The correct answer is: C
Explanation:
Assisting users with an assessment of the government’s operating results, level of services provided, and the ability to meet cash requirements is a broad principle and is the second objective outlined in Concepts Statement No. 1. While assisting users in understanding funding objectives for public retirement systems and the potential tax burden on future taxpayers may be one of the significant elements for any governmental financial report, it is not specifically discussed as a reporting objective. Assisting users in identifying management’s policies for selection of critical programs and services to provide and the annual tax burden to finance those programs is an example of a specific reporting issue, not a broad principle. Assisting users in determining the annual tax burden per capita for each of the major programs provided by the governmental entity is an example of specific information that is critical to users, but not discussed as a primary broad objective.
Arlen City’s fiduciary funds contained the following cash balances at December 31:
Under the Forfeiture Act-cash confiscated from illegal activities;
principal can be used only for law enforcement activities $300,000
Sales taxes collected by Arlen to be distributed
to other governmental units 500,000
What amount of cash should Arlen report in its permanent funds at December 31?
A. $0 B. $300,000 C. $500,000 D. $800,000
The correct answer is: A
Explanation:
The cash collected under the For feiture Act, can only be used for law enforcement activities. It should be accounted for in a special revenue fund because the Act does not require the preservation of fund principal and the principal may be used for Arlen’s benefit . An agency fund is established to account for assets received by a government in its capacity as an agent for individuals, businesses, or other governments. Therefore, the sales taxes collected by Arlen to be distributed to other governments are accounted for in an agency fund.
For a municipal solid waste landfill, equipment and facilities included in the estimated total current cost of closure and postclosure care are reported in the statement of net position as which of the following?
A. Current expenses B. Capital assets C. A reduction of the liability D. None of the above
The correct answer is: C
Explanation:
Equipment and facilities included in the estimated total current cost of closure and postclosure care are not reported as current expenses, but as a reduction of the liability. A liability that would be excluded from the “capital-related debt” portion of the statement of net position is the liability for closure and postclosure care associated with a municipal solid waste landfill.
Palm City acquired, through forfeiture as a result of nonpayment of property taxes, a parcel of land that the city intends to use as a parking lot for general governmental purposes. The total amount of taxes, liens, and other costs incurred by Palm incidental to acquiring ownership and perfecting title was $20,000. The land’s fair market value at the forfeiture date was $60,000. What amount should be reported in the governmental activities column of the governmentwide financial statements for this land?
A. $0 B. $20,000 C. $60,000 D. $80,000
The correct answer is: B
Explanation:
General fixed assets acquired by foreclosure are recorded at the lower of (1) the amount due for taxes, special assessments, penalties and interest, plus foreclosure costs or (2) appraised fair market value. Therefore, since the $20,000 Palm incurred for taxes, liens, and other costs incidental to acquiring ownership and perfecting title is less than the land’s $60,000 fair market value at the forfeiture date, the land should be reported in the governmental activities column of government-wide financial statements at $20,000.
Governments are required to present budgetary comparisons for which of the following?
A. The general fund only B. The general fund and all major special revenue funds C. All governmental and proprietary funds D. None of the above
The correct answer is: B
Explanation:
Governmentsare required to present budgetary comparisons for the general fund and all major special revenue funds in the general purpose external financial reports, but not for proprietary funds.
Lily City uses a pay-as-you-go approach for funding postemployment benefits other than pensions. The city reports no other postemployment benefits (OPEB) liability at the beginning of the year. At the end of the year, Lily City reported the following information related to OPEB for the water enterprise fund:
Benefits paid $100,000 Annual required contribution 500,000 Unfunded actuarial accrued liability 800,000What amount of expense for OPEB should Lily City’s water enterprise fund report in its fund level statements? A. $100,000 B. $500,000 C. $600,000 D. $1,400,000
The correct answer is: B
Explanation:
Other postemployment benefits (OPEB) are typically in the form of health care coverage. Most entities fund their OPEB on a pay-as-you-go basis, paying only the amount equal to benefits claimed. GASB believes the pay-as-you-go approach does not reflect the true OPEB liability, and issued GASB 45 outlining OPEB reporting and disclosure requirement. The objective is to systematically and rationally allocate the present value of the projected cost of benefits over the working life of benefiting employees. This allocation is done with an actuarial valuation. The amount allocated to a given year is known as the annual required contribution (ARC), which is the basic amount of expense (i.e., $500,000). The present value of benefits earned to date by employees is known as the actuarial accrued liability. The difference between this amount and any resources placed in trust is known as the unfunded actuarial accrued liability.
The debt service transactions of a special assessment bond issue for which the government is not obligated in any manner should be reported in the
A. Custodial Fund B. Enterprise Fund C. Special Revenue Fund D. Long-Term Debt Account Group
The correct answer is: A
Explanation:
The correct answer is (A).
Although the special assessment debt for which the government is not obligated in any manner is not reported as government debt, the government usually acts as a debt service agent for the special assessment district.
The government has a fiduciary responsibility to collect the special assessments and to remit the collections to the bondholders when the debt service payments come due.
However, if collections are insufficient to cover required debt service payments, the government is not obligated to pay the difference and does not intend to do so.
Thus, the government is acting in a custodial capacity with respect to the debt service transactions, and these transactions should be accounted for in a Custodial Fund.
King City Council will be establishing a library fund. Library fees are expected to cover 55% of the library’s annual resource requirements. King has decided that an annual determination of net income is desirable in order to maintain management control and accountability over library. What type of fund should King establish in order to meet their measurement objectives?
A. Special revenue fund. B. General fund. C. Internal service fund. D. Enterprise fund.
The correct answer is: D
Explanation:
Enterprise funds must be used to account for a government’s business-type operations that are financed and operated like private businesses—where the government’s intent is that all costs of providing goods or services to the general public on a continuing basis are to be financed or recovered primarily through users charges. Governments are permitted to account for virtually any type of self-contained business-type activity in enterprise funds if it prefers to do business-type accounting rather than general government accounting. An enterprise fund is a proprietary fund.
Which of the following would be reported as program revenues on a local government’s government-wide statement of activities?
A. Charges for services B. Taxes levied for a specific function C. Proceeds from the sale of a capital asset used for a specific function D. Interest revenues
A.
Charges for services
Explanation:
On the government-wide statement of activities, the net revenue (expense) format and net program cost format are used. Charges for services are one of the program revenues reported on a local government’s government-wide statement of activities. Taxes levied, proceeds from the sale of a capital asset, and interest revenues would fall under general revenues.
Economic and demographic information in the statistical section of the CAFR helps users do which of the following?
A. Gauge the resources for the government’s revenues and demands for public services B. Calculate the debt burden per capita C. Evaluate the direct and overlapping debt burden on the available taxpayers D. Perform all of the above actions
The correct answer is: A
Explanation:
Economic and demographic information can help users gauge the economic health of a community, and thus, the demands for governmental services as well as the key factors that will impact revenues. The debt burden per capita is presented in the debt capacity tables. Overlapping debt information is presented in the debt capacity tables.
Cy City’s Municipal Solid Waste Landfill Enterprise Fund was established when a new landfill was opened January 3, year 1. The landfill is expected to close December 31, year 20. Cy’s year 1 expenses would include a portion of which of the year 21 expected disbursements?
Cost of a final cover to be applied to the landfill. Cost of equipment to be installed to monitor methane gas buildup. A. I only. B. II only. C. Both I and II. D. Neither I nor II.
The correct answer is: C
Explanation:
In accounting for municipal solid waste landfill (MSWLF) costs, a portion of the estimated total current costs must be recognized as an expense and as a liability in each period during which the MSWLF accepts solid waste. The estimated total current costs of MSWLF closure and postclosure care should include those costs which result in disbursements near, or after, the date that the MSWLF stops accepting solid waste and during the postclosure period, including the cost of a final cover and monitoring equipment.
Which of the following is considered the most prevalent fiduciary fund?
A. Custodial fund B. Pension trust fund C. Investment trust fund D. Private-purpose trust fund
The correct answer is: B
Explanation:
The most prevalent fiduciary fund is the pension trust fund, not the custodial fund, the investment trust fund, or the private-purpose trust fund.
Which of the following statements about the governmental statement of activities is false?
A.
The statement of activities highlights the costs of programs and the sources of revenues for those programs.
B.
Governmental services will likely show large deficits in the net revenue column.
C.
Tax revenues may be reported as program income if these resources are restricted to a specific program.
D.
Governments are allowed to allocate administrative and overhead costs from the general government category to individual programs.
The correct answer is: C
Explanation:
All taxes are considered general revenues, even if restricted to a specific program. GASB Statement No. 34 shifted the governmental operating statement to a program focus that first matches the costs of program revenues and the sources of revenues for those programs. The statement of activities divides the operations of the government not into traditional line items—but into program or operational categories. Governmental services will likely show large deficits in the net (expense) revenue column. This allocation is optional, but governments are allowed to allocate the administrative and overhead costs from the general government category.
Which type of fund is allowed to incur long-term debt through the issuance of bonds?
A. Governmental funds B. Proprietary funds C. Both governmental and proprietary funds D. Neither governmental nor proprietary funds
The correct answer is: C
Explanation:
Both governmental and proprietary funds could issue long-term debt in the form of bonds payable. The differences would lie in the reporting of the bond proceeds and the related liabilities on the financial statements.
Carlson City’s fiscal year ends December 31. On August 1, the city issued a purchase order for new vehicles to be delivered at the rate of two per month beginning October 15. Twelve vehicles were delivered as scheduled and payments of $264,000 were made upon delivery. If these were the only transactions made by the city, which of the following balances would appear on the balance sheet as of December 31?
A Encumbrances Reserve for encumbrances $132,000 132,000 B Unassigned fund balance Reserve for encumbrances $132,000 132,000 C Reserve for encumbrances Unassigned fund balance $264,000 264,000 D Encumbrances Reserve for encumbrances $264,000 264,000
The correct answer is: B
Explanation:
The correct answer is (B).
Year-end closing entries close the budgetary accounts for open encumbrances (i.e., orders that were placed during the year but not fulfilled as of the end of the year).
Also, unreserved fund balance on the books is reduced since an encumbrance represents a legal commitment to purchase.
Between October 15 and December 31, six vehicles were delivered at the rate of two per month, while an encumbrance was created for 12 vehicles at $264,000.
Therefore, $132,000 was paid for the delivery of six vehicles.
Dr: Unassigned Fund Balance $132,000
Cr: Reserved for encumbrances $132,000
Note: The schedule was for two vehicles per month starting October 15 for a total of 12 vehicles. As all vehicles were delivered as planned, two vehicles each for October, November, and December will total to six vehicles.
It is inappropriate to record depreciation expense in the government-wide financial statements related to the assets in which of Kellick City’s funds?
A. Custodial fund B. Enterprise fund C. General fund D. Special revenue fund
The correct answer is: A
Explanation:
The correct answer is (A)
Custodial funds should be used to report resources held by the reporting government in a purely custodial capacity (assets equal liabilities).
These funds typically involve only the receipt, temporary investment, and remittance of fiduciary resources to individuals, private organizations, or other governments.
Custodial funds generally have neither capital assets nor expenses, and as such would not have depreciation expense
The following information pertains to property taxes levied by Oak City for the calendar year 1:
Collections during year 1 $500,000
Expected collections during the first 60 days of year 2 100,000
Expected collections during the balance of year 2 60,000
Expected collections during January year 3 30,000
Estimated to be uncollectible 10,000
Total levy $700,000What amount should Oak report for year 1 net property tax revenues?
A.
$700,000
B.
$690,000
C.
$600,000
D.
$500,000
The correct answer is: C
Explanation:
Governmental funds use the modified accrual basis of accounting, under which revenues susceptible to accrual (e.g., property taxes) are recognized when they become measurable and available for use. ‘Available for use’ means that the revenues will be collected within the current period or collected early enough in the next period (i.e., within 60 days or so) to be used to pay for expenditures incurred in the current period. Therefore, for year 1, Oak should report property tax revenues of $600,000, the sum of the of property taxes levied and collected in year 1 and the expected property tax collections during the first 60 days of year 2.
Polk County’s solid waste landfill operation is accounted for in a governmental fund. Polk used available cash to purchase equipment that is included in the estimated current cost of closure and post-closure care of this operation. How would this purchase affect the long-term asset and the long-term liability amounts in Polk’s general fund?
Asset Liability A Increase Decrease B Increase No effect C No effect No effect D No effect Decrease
The correct answer is: D
Explanation:
The correct answer is (D).
A MSWLF (Municipal Solid Waste Landfill) is accounted for in a government-type fund and should accrue the cost of assets required for closure and post-closure on an annual basis with a corresponding liability. When the available cash is used to purchase equipment, it has no effect on the asset but the liability would be reduced.
Equipment, facilities, services, and final expenses included in the estimated total current cost of closure and post-closure care should be reported as a reduction of the reported liability for closure and post-closure care when they are acquired.
Which of the following funds of a local government would report transfers to other funds as an other financing use?
A. Enterprise B. Internal service C. Pension trust D. General
The correct answer is: D
Explanation:
The correct answer is (D).
Interfund transactions appear in the operating statements of the affected funds. In governmental funds, transfers are reported as Other Financing Sources (Uses). In proprietary funds, transfers should be reported as Transfers In (Out). Only the general fund is a governmental fund and would report transfers to other funds as an other financing use. The enterprise and internal service funds are proprietary funds that would report transfers to other funds as transfers out. The pension trust fund is a fiduciary fund that is accounted for in essentially the same manner as proprietary funds.
Which of the following is NOT included on the face of the government-wide statement of net position?
A. Governmental activities B. General fund C. Business-type activities D. Component unit
The correct answer is: B
Explanation:
The three primary columns shown on the face of the government-wide statement of net position are governmental activities, business-type activities, and component units if applicable. The balances for the general fund would be shown on the balance sheet of the governmental fund
Baker City is planning to lease some trucks. Determine the lease term from the information given below:
Non-cancellable period of the lease: 5 years
Renewal periods not certain to be exercised: 2 years
Periods covered under a termination option: 1 year
Non-termination periods controlled by the lessor: 2 years
It is to be assumed that the city won’t exercise the termination option.
A. 5 years B. 10 years C. 8 years D. 7 years
The correct answer is: C
Explanation:
The correct answer is (C).
For Governmental Accounting, the lease term is calculated the same way as it is in Financial Accounting under the FASB.
The lease term or tenure of the lease is calculated as follows:
Non-cancellable period: 5 years.
Renewal periods “reasonably certain” of exercise
Periods covered by a termination option reasonably certain not to be exercised: 1 Year
Renewal periods (non-termination periods) controlled by lessor: 2 Years.
Total: 8 Years.
In the given case of Baker City, all periods are included except for the renewal period which is not certain to be exercised.
Which of the following is the paramount objective of financial reporting by state and local governments? A Reliability B Consistency C Comparability D Accountability
D
Explanation:
Governments are focused on programs and services that will improve the quality of life in their communities. Governmental entities must provide accountability for how they raised resources for those programs and explain how they managed and safeguarded those public resources.
Encumbrances would not appear in which fund?
A Capital projects. B Special revenue. C General. D Enterprise.
D
Explanation:
The encumbrance system is used in governmental funds (general, special revenue, and capital projects funds) to prevent over-expenditure and to demonstrate compliance with legal requirements.The enterprise fund is a proprietary fund and does not use the encumbrance system.Option (a), (b) and (c) are incorrect because these funds can have encumbrances.
Oak County incurred the following expenditures in issuing long-term bonds:
Issue cost $ 400,000
Debt insurance 90,000
When Oak establishes the accounting for operating debt service, what amount should be deferred and amortized over the life of the bonds?
A $90,000 B $0 C $400,000 D $490,000
Explanation:
The correct answer is (B).
The general long-term debt of a state or local government is reported:
Government-wide financial statements: Accrual basis of accounting.
Governmental Funds (Debt Service Funds): Modified Accrual basis of accounting.
In Government Funds (Debt Service Funds) under Modified Accrual basis of accounting costs are recognized using the expenditure principle, and recorded in the period the obligation to pay arises, irrespective of when the goods/services are being actually used by the government entity.
Accordingly, the modified accrual basis does not allow for amortization of issue costs and debt insurance premiums, which must be recognized in full when the liability is incurred.
The amount deferred and amortized over the life of the bonds is $0.
$490,000 is expensed in the period incurred.
Dale Town’s public school system is administered by a separately elected board of education. The board of education is not organized as a separate legal entity and does not have the power to levy taxes or issue bonds. Dale’s town council approves the school system’s budget. Where should Dale report the public school system in its government-wide information?
A Within the component units column B Within the governmental activities column C In the notes to the financial statements D In the required supplementary information
Explanation:
The correct answer is (B).
The government-wide statements should display information about the reporting government as a whole, except for its fiduciary activities. Governmental-Wide Financial Statement’s, Statement of Net Position has four columns as follows:
Governmental activities: Presents consolidated results of all the Governmental Funds and the Internal Service Funds including all component units subject to blended presentation
Business-type activities: Presents consolidated results of all Enterprise Funds
Total: Presents a total of Governmental Activities & Business-type Activities
Component units: Presents results of all the component units subject to discrete presentation
Component units are legally separate organizations for which elected officials of the primary government are financially accountable.
A unit classifies for a discreet presentation (within the component unit’s column) when it meets the following criteria or else it is presented as a blended component within the Governmental activities column.
Services provided by the government are not primary to the government
Management of the unit consists of separately elected officials i.e. a separately elected governing board.
Separate Legal Entity
Fiscally Independent i.e. power to determine its own budget, levy taxes, and issue bonds.
Dale Town’s public-school system has separately elected governing board but it is not a separate legal entity and does not have the power to levy taxes or issue bonds (i.e. not fiscally independent) and hence it fails the criteria for discreet presentation and will be presented as a blended component within Governmental activities column.
In governmental accounting, a fund is
The basic accounting unit. Used to assist in ensuring fiscal compliance. A I only B II only C Both I and II D Neither I nor II
C
Explanation:
A specific governmental unit is not accounted for through a single accounting entity. Instead, the accounts of a government are divided into several funds. A fund is a fiscal and accounting entity with a self-balancing set of accounts recording cash and other financial resources, together with all related liabilities and residual equities and balances, and changes therein, which are segregated for the purpose of carrying on specific activities or attaining certain objectives in accordance with special regulations, restrictions, or limitations.
As described in GASB Concepts Statement No. 3, the distinction between required supplementary information (RSI) and other supplementary information (OSI) is generally based on which of the following?
A
Whether the GASB has specified the requirement to present the information
B
Whether the government has decided to present the information
C
The public’s demand for information from the government
D
None of the above
A
Explanation:
The distinction between RSI and OSI is generally based on whether the GASB has specified the requirement to present the information, not the public’s demand for the information. Management can certainly add to RSI, but additional information must be presented separately so that users can consistently compare reports among governmental entities.
The governmental category in the fund statements includes which of the following?
A
General fund, special revenue, debt service, and enterprise funds
B
General fund, special revenue, debt service, capital projects, and permanent funds
C
General fund, special revenue, fiduciary funds, capital projects, and internal service funds
D
General fund, special revenue, debt service, and internal service funds
B
Explanation:
The governmental category in the fund statements summarizes those programs funded with general types of revenues (generally taxes and grants). This category includes the general fund, special revenue, debt service, capital projects, and permanent funds. Fiduciary funds are not available to support the government’s own programs. Proprietary funds include: enterprise funds and internal service funds.
Cedar City issues $1,000,000, 6% revenue bonds were issued at par on April 1, to build a new water line for the water enterprise fund. Interest is payable every six months. What amount of interest expense should be reported for the year ended December 31?
A $0 B $30,000 C $45,000 D $60,000
C
Explanation:
Since this is an enterprise fund, accrual accounting applies. Interest expense for the year includes the $30,000 ($1,000,000 × 6% × 1/2 year) paid on October 1st and the $15,000 ($1,000,000 × 6% × 1/4 year) accrued expense from October 1st through December 31 for a total of $45,000.
Marta City's school district is a legally separate entity but two of its seven board members are also city council members and the district is financially dependent on the city. The school district should be reported as a A Blended unit. B Discrete presentation. C Note disclosure. D Primary government.
B
Explanation:
Component units should be presented discretely unless either (a) the components unit’s governing body is substantively the same as the governing body of the primary government, or (b) the component unit provides services almost entirely to the primary government, or almost exclusively benefits the primary government although it does not provide services directly to it.
Elliott Township purchased a firetruck costing $1,000,000 in its general fund on January 1 of the current year. The truck had a useful life of 10 years and the Township has a policy of taking a full year’s depreciation in the first year. How much depreciation related to the truck would the Township report on its statement of activities?
A $1,000,000 B $100,000 C $10,000 D $0
B
Explanation:
The statement of activities is a government-wide financial statement which uses the accrual method of accounting. The fire truck would be reported as a capital asset on the government-wide statement of net position, and $100,000 in depreciation would be reported on the statement of activities.
Which of the following statements is correct concerning disclosure of reverse repurchase and fixed coupon reverse repurchase agreements?
A
Related assets and liabilities should be netted.
B
Related interest cost and interest earned should be netted.
C
Credit risk related to the agreements need not be disclosed.
D
Underlying securities owned should be reported as “Investments.”
D
Explanation:
The underlying securities owned with regard to reverse repurchase and fixed coupon reverse repurchase agreements should be reported as “Investments.” Related assets and liabilities and interest cost and interest earned should not be netted. Credit risk related to such agreements must be disclosed.
Which of the following amounts are included in a general fund’s encumbrance account?
Outstanding vouchers payable amounts Outstanding purchase order amounts Excess of the amount of a purchase order over the actual expenditure for that order A I only. B I and III. C II only. D II and III.
C
Explanation:
The encumbrance system is used by governmental funds to prevent overexpenditure and to demonstrate compliance with legal requirements. When a purchase order is issued, the estimated amount of the planned expenditure is encumbered by debiting Encumbrances and crediting Fund Balance Reserved for Encumbrances. When the related invoice is received, the encumbrance entry is reversed and the actual expenditure is recorded. Thus, the balance of the Encumbrance account will always equal the outstanding purchase order amounts until the books are closed at year-end.
Which of the following funds should be reported as part of local government’s governmental activities column in its government-wide statements?
A Debt service B Custodial C Private-purpose trust D Pension trust
Explanation:
The correct answer is (A).
The debt service fund should be reported as part of local government’s governmental activities column in its government-wide statements. Reporting capital assets and long-term liabilities are required in the government-wide statements. Custodial, private-purpose trust, and pension trust funds are all fiduciary funds. Fiduciary activities are not included in the government-wide statements because the assets and liabilities cannot be used to support the government’s own programs.
Option (b), (c) and (d) are incorrect because they are fiduciary funds and excluded from government-wide F/S.
Pine City's year end is June 30. Pine levies property taxes in January of each year for the calendar year. One-half of the levy is due in May and one-half is due in October. Property tax revenue is budgeted for the period in which payment is due. The following information pertains to Pine's property taxes for the period from July 1, year 1, to June 30, year 2: Calendar year Year 1 Year 2 Levy $2,000,000 $2,400,000 Collected in: May $950,000 $1,100,000 July 50,000 60,000 October 920,000 December 80,000 The $40,000 balance due for the May, year 2, installments was expected to be collected in August, year 2. What amount should Pine recognize for property tax revenue for the year ended June 30, year 2? A $2,160,000 B $2,200,000 C $2,360,000 D $2,400,000
B
Explanation:
Governmental funds use the modified accrual basis of accounting, under which, revenues are recognized when they become measurable and available for use. ‘Available for use’ means that the revenues will be collected within the current period or collected early enough in the next period (e.g., within 60 days or so) to be used to pay for expenditures incurred in the current period. Therefore, for the year ended June 30, year 2, Pine should recognize property tax revenue of $2,200,000. This amount is comprised of (1) $1,000,000 (i.e., $920,000 + $80,000) of property taxes levied in January of year1 which were collected in October and December of year 1 and (2) $1,200,000 (i.e., $1,100,000 + $60,000 + $40,000) of property taxes levied in January year 2 which were collected in May, July, and August year 2.
During the current year Knoxx County levied property taxes of $2,000,000, of which 1% is expected to be uncollectible. The following amounts were collected during the current year:
Prior year taxes collected within the 60 days of the current year $ 50,000
Prior year taxes collected between 60 and 90 days into current year 120,000
Current year taxes collected in the current year 1,800,000
Current year taxes collected within the first 60 days of the subsequent year 80,000What amount of property tax revenue should Knoxx County report in its entity-wide statement of activities?
A
$1,800,000
B
$1,970,000
C
$1,980,000
D
$2,000,000
C
Explanation:
Entity-wide statements, the government-wide statements, are done under normal accrual accounting. Only fund financial statements, done under modified accrual, would take into account any 60-day criteria. Under normal accrual accounting the amount of property tax revenue reported would be the $2,000,000 property taxes levied less the 1% ($20,000) for estimated uncollectible taxes for a total of $1,980,000. The entry would be:
Taxes receivable - current 2,000,000
Estimated uncollectible taxes 20,000
Revenues control 1,980,000To record property tax revenue.
Which of the following would be reported as program revenues on a local government's government-wide statement of activities? A Charges for services B Taxes levied for a specific function C Proceeds from the sale of a capital asset used for a specific function D Interest revenues
A
Explanation:
On the government-wide statement of activities, the net revenue (expense) format and net program cost format are used. Charges for services are one of the program revenues reported on a local government’s government-wide statement of activities. Taxes levied, proceeds from the sale of a capital asset, and interest revenues would fall under general revenues.
A city government would report each of the following categories in its government-wide statement of net position except
A Governmental activities. B Business-type activities. C Fiduciary activities. D Component units.
Explanation:
The correct answer is (C).
In the government-wide financial statements, all government and proprietary funds are included (but not fiduciary funds) along with discreetly presented component units in the statement of net position.
(A), (B) and (D) are incorrect because these are included in the statement of net position.
On January 1, Fonk City approved the following general fund resources for the new fiscal period:
Property taxes $ 5,000,000 Licenses and permits 400,000 Intergovernmental revenues 150,000 Transfers in from other funds 350,000 What amount should Fonk record as estimated revenues for the new fiscal year?
A $5,400,000 B $5,550,000 C $5,750,000 D $5,900,000
B
Explanation:
The correct answer is B. Estimated revenues are revenues that are expected to be available to be spent in the period.
Ref
Summary
Amount
a
Property taxes
$5,000,000
b
Licenses & Permits
$400,000
c
Inter-governmental revenues
$150,000
d
Estimated Revenue (a+b+c)
$5,550,000
Transfers in from other funds are not considered because it is an estimated other financing source, not revenue. Inter-governmental revenue is included in the calculation of estimated revenue. Transfers in from other funds of $350,000 are not included in the calculation for estimated revenue.
Which of the following is the measurement focus and basis of accounting for the government-wide financial statements?
Measurement Focus Basis of accounting A Economic resources Accrual B Economic resources Modified accrual C Current financial resources Accrual D Current financial resources Modified accrual
A
Explanation:
(a) GASB 34 requires the economic resources measurement focus and accrual basis of accounting for all amounts in the government-wide financial statements. Option (b) and (c) are incorrect as per the above explanation. Option (d) is incorrect because it applies to governmental funds; not to government-wide F/S.
A city government reported a $9,000 increase in net position in the motor pool internal service fund, a $12,000 increase in net position in the water enterprise fund, and a $7,000 increase in the employee pension fund. The motor pool internal service fund provides service primarily to the police department. What amount should the city report as the change in net position for business-type activities in its statement of activities?
A $9,000 B $12,000 C $21,000 D $28,000
Explanation:
The correct answer is (B).
Business type activities that are normally financed through user charges and reports the consolidated results of all enterprise funds. It does not include internal service funds, which are accounted for in governmental activities. The employee pension fund is a fiduciary fund and not included in business-type activities. The $12,000 increase in net position from Water enterprise is included.
(A) is incorrect because the motor pool internal service fund is excluded from business-type activities.
(C) is incorrect because the calculation also includes motor pool internal service fund which should be excluded.
(D) is incorrect because the motor pool Internal service fund and employee pension trust fund should be excluded from business-type activities.
Lake County received the following proceeds that are legally restricted to expenditure for specified purposes:
Levies on affected property owners to install sidewalks $500,000
Gasoline taxes to finance road repairs 900,000
What amount should be accounted for in Lake’s special revenue funds?
A $1,400,000 B $900,000 C $500,000 D $0
B
Explanation:
Special revenue fund sources includes fees, grants, specific taxes, and other earmarked revenue sources. The gasoline taxes to finance road repairs for $900,000 would be a specific tax, accounted under special revenue funds. Levies on affected property owners to install sidewalks is usually accounted for under the agency fund. It may also be accounted for in a debt service fund, for example: the levies are used to repay a bond that was issued to install the sidewalks. Options (a), (c) and (d) are incorrect based on the above explanation.
Grove Township issued $50,000 of bond anticipation notes at face amount in the current year and placed the proceeds into its capital projects fund.All legal steps were taken to refinance the notes, but Grove was unable to consummate refinancing. In the capital projects fund, what account should be credited to record the $50,000 proceeds?
A Other Financing Sources Control B Revenues Control C Deferred Revenues D Bond Anticipation Notes Payable
D
Explanation:
A governmental unit would issue bond anticipation notes (BANs) to provide funds to defray costs expected to be incurred before the related bonds are issued. Such notes are treated as long-term debt, even if due within one year, if (1) they are to be repaid with the proceeds of the bond issue, (2) all legal steps have been taken to refinance the notes, and (3) the intent is supported by an ability to refinance the short-term notes on a long-term basis. Since all of these criteria are not met for the bond anticipation notes in question, they are reported as a liability of the capital projects fund.
Which of the following funds of a local government would report transfers to other funds as another financing use?
A Enterprise. B Internal service. C Pension trust. D General.
Explanation:
The correct answer is (D).
The general fund of a local government is used to report transfers to other funds as an “other financing use.”
Only governmental funds (general, special revenue, debt service, capital projects, and permanent) report transfers to other funds as “other financing use.”
Chase City uses an internal service fund for its central motor pool. The assets and liabilities account balances for this fund that are not eliminated normally should be reported in the government-wide statement of net position as A Governmental activities B Business-type activities C Fiduciary activities D Note disclosures only
A
Explanation:
Although internal service funds are proprietary funds, they appear in the government-wide statements as governmental activities in the new reporting model.
A special-purpose government is distinguished from a general purpose government by which of the following?
A
It is established by a state government.
B
It provides limited services, such as only governmental or business-type activities.
C
It does not manage any fiduciary funds.
D
It is not governed by a separately elected legislative body.
B
Explanation:
The distinction between general purpose and special-purpose governments is based on the entity’s programs and services, not how the government was created. Special-purpose governments only provide limited services; unlike cities, counties, states, towns or villages that provide a broad range of services. It is possible that a special purpose government may only manage fiduciary programs. Governance structure is not the distinguishing feature of special-purpose governments.
The water department of a city provides water to the nearby districts for a monthly fee. In year 2, it decides to take machinery on lease from one of the firms in the city. The lease term was decided to be 3 years and the ownership of the machinery would not transfer to the department at the end of the term. Given that the useful life of the machinery was 2 years and the value of the leased asset was $120,000 in the books of the water department, what would be the amortization in the books in year 2?
A $60,000 B $40,000 C $120,000 D $0
Explanation:
The correct answer is (A).
In leases other than a short-term lease and contracts that transfer ownership, the lessee recognizes a right-to-use asset in its books. The asset is required to be amortized over the life of the asset or the lease term, whichever is less. In the given question, the useful life of 2 years is less than the lease term of 3 years. The value of the ROU asset shall be amortized over a period of 2 years.
Annual amortization = 120,000/2 = $60,000
The following obligations were among those reported by Fern Village at December 31:
Vendor financing with a term of 10 months when incurred,
in connection with a capital asset acquisition that is not
part of a long-term financing plan $ 150,000
Long-term bonds for financing of capital asset acquisition 3,000,000
Bond anticipation notes due in six months, issued as part
of a long-term financing plan for capital purposes 400,000What aggregate amount should Fern report as general long-term capital debt at December 31?
A
$3,000,000
B
$3,150,000
C
$3,400,000
D
$3,550,000
C
Explanation:
Fern should report $3,400,000 as long-term capital debt. This amount is comprised of (1) $3,000,000 of long-term bonds for financing of capital asset acquisition and (2) $400,000 of bond anticipation notes due in six months that were issued as part of a long-term financing plan for capital purposes. Although the bond anticipation notes (BANs) are due in six months, the intent is to refinance the bond anticipation notes on a long-term basis. The vendor financing is not reported as long-term capital debt because it is not part of a long-term financing plan.
Which format must an enterprise fund use to report cash flow operating activities in the statement of cash flows?
A
Indirect method, beginning with operating income.
B
Indirect method, beginning with change in net position.
C
Direct method.
D
Either direct or indirect method.
C
Explanation:
An enterprise fund is one of the governmental proprietary funds. Governments should present a statement of cash flows for proprietary funds based on the provisions of GASB statements. The GASB states the direct method of presenting cash flows from operating activities (including a reconciliation of operating cash flows to operating income) should be used.
Which event(s) should be included in a statement of cash flows for a governmental entity?
Cash inflow from issuing bonds to finance city hall construction. Cash outflow from a city utility representing payments in lieu of property taxes. A I only B II only C Both I and II D Neither I nor II
Explanation:
The correct answer is (B).
Bonds issued to finance city hall construction is recorded in its Capital Projects Fund, which is a Governmental Fund. Governmental Funds are accounted on a modified accrual basis and a Statement of Cash Flows is not presented for Governmental Funds.
Under governmental accounting, a Statement of Cash Flows is prepared only for Proprietary Funds.
A cash flows statement includes receipts and payments resulting only from the Proprietary Fund’s activities. The payments in lieu of property taxes are from an operating activity of an Enterprise Fund (Proprietary).
Nox City reported a $25,000 net increase in the fund balances for total governmental funds. Nox also reported an increase in net position for the following: Motor pool internal service fund $9,000 Water enterprise fund 12,000 Employee pension fund 7,000The motor pool internal service fund provides service to the general fund departments. What amount should Nox report as the change in net position for governmental activities? A $25,000 B $34,000 C $41,000 D $46,000
Explanation:
The GWS governmental activities column includes the governmental funds plus most internal service funds and capital assets not specific to business-type activities or fiduciary funds. $25,000 + $9,000 = $34,000. The water enterprise fund is a business-type activity. The employee pension fund is a fiduciary fund; it doesn’t appear in the GWS.
Which fund would report the amount of bonds payable that are due within one year?
A Governmental funds B Proprietary funds C Both governmental and proprietary funds D Neither governmental nor proprietary funds
B
Explanation:
Proprietary funds report the current amounts due for any bonds outstanding on its statement of net position. At the fund level, governmental funds do not report the current portions of principal or interest due.
Which of the following is a minimum required report for the basic financial statements of a government entity?
A Fund financial statements. B Management's discussion and analysis. C Required supplementary information. D Budgetary comparison schedules.
Explanation:
The correct answer is (A).
The various elements of the basic financial statements of a government entity are:
Government-Wide Financial Statements
Fund Financial Statements
Notes to the Financial Statements
Hence, out of the given options, fund financial statements qualifies as the minimum required report for the basic financial statements of a government entity.
Programs funded with taxes, grant revenues, and other nonexchange resources are reported in the government-wide statements as which of the following? A Social service activities B Intergovernmental activities C Governmental activities D Business-type activities
C
Explanation:
Governmental activities include those programs funded with taxes, grant revenues, and other nonexchange resources (not social service activities or intergovernmental activities). Business-type activities are financed in whole or in part by fees charged to external parties for goods and services. Enterprise funds are used to account for these programs.
For which of the following funds do operating transfers affect the results of operations?
Governmental funds Proprietary funds A No No B No Yes C Yes Yes D Yes No
C
Explanation:
Operating transfers should be recorded in Other Financing Sources Uses or Other Financing accounts and reported after revenues and expenditures or expenses, but before determining the results of operations in the operating statement.
Eureka City should issue a statement of cash flows for which of the following funds?
Eureka City Hall Capital Projects Fund Eureka Water Enterprise Fund A No Yes B No No C Yes No D Yes Yes
A
Explanation:
The capital projects fund is a governmental fund, whereas the enterprise fund is a proprietary fund. A statement of cash flows should be presented for all proprietary funds, but not for governmental funds.
The following information for the year ended June 30 pertains to a proprietary fund established by Glen Village in connection with Glen’s public parking facilities:
Receipts from users of parking facilities $600,000
Expenditures: Parking meters 410,000
Expenditures: Salaries and other cash expenses 96,000
Depreciation of parking meters 94,000
For the year ended June 30, this proprietary fund should report net income of
A $0 B $94,000 C $96,000 D $410,000
D
Explanation:
Proprietary funds account for their fixed assets in the same manner as commercial enterprises; therefore, the expenditure for the parking meters should be recorded in the fund’s fixed asset accounts. Enterprise funds and internal service funds are the two types of proprietary funds. The fund in question is an enterprise fund because it is a self-supporting fund which provides goods and/or services to the general public.
Parking meter is an so its a fixed asset
Net Income = 600k -96k-94k
=410k
A local government has the following liabilities on its adjusted trial balance at the end of the current year. What amount would the government report on its balance sheet as total liabilities? $50,000 accrued payroll $100,000 bonds payable – current portion $900,000 bonds payable – long-term portion $25,000 accounts payable A $1,075,000 B $175,000 C $100,000 D $75,000
D
Explanation:
The balance sheet is a fund level financial statement and thus uses the modified accrual method of accounting. This means that the $1,000,000 related to the bonds will not be reported, and the answer is the $50,000 in accrued payroll and the $25,000 in accounts payable.
Land and other real estate held as investments by endowments in a government’s permanent fund should be reported at A Historical cost B The lower of cost and net realizable value C Fair value D Fair value less costs of disposal
C
Explanation:
Endowments are donations and donations are recorded as revenue at fair value, regardless of their cost to the donor, and reported at fair value at statement dates.
Property taxes and fines represent which of the following classes of nonexchange transactions for governmental units? A Derived tax revenues B Imposed nonexchange revenues C Government-mandated nonexchange transactions D Voluntary nonexchange transactions
B
Explanation:
Imposed nonexchange revenues are assessments on non-governmental entities, and include property taxes and fines or forfeitures.
Which of the following can be reported in a capital projects fund?
A
Bond proceeds for sewer treatment plant upgrades
B
Committed resources to expand water distribution system
C
Restricted, committed, or assigned resources for governmental capital outlays
D
Assigned resources for landfill postclosure care
C
Explanation:
Restricted, committed, or assigned resources for governmental capital outlays are reported in a capital projects fund. Sewer services, water utilities, and landfills are usually reported in an enterprise fund because these services are funded with user fees. Enterprise fund capital assets are not reported in a capital projects fund.
Tang City received land from a donor who stipulated that the land must remain intact, but any income generated from the property may be used for general government services. In which fund should Tang City record the donated land?
A Special revenue. B Permanent. C Private-purpose trust. D Agency.
Explanation:
The correct answer is (B).
The Permanent Fund accounts and reports assets whose principal is restricted and may not be spent, but must be invested on a permanent basis. Since land is restricted by the donor and income generated from the land can be spent, Tang city would record in permanent fund.
Option (a) is incorrect because special revenue fund accounts for and reports specific revenues from earmarked sources that are restricted or committed to be used to finance designated activities other than capital projects and debt service.
Option (c) is incorrect because private-purpose trust fund accounts for resources that are being held for benefit of private persons, organizations or other governments.
Option (d) is incorrect because agency funds accounts for collected amounts that must be transferred to other funds or outsiders.
The measurement focus of governmental-type funds is on the determination of
Flow of financial resources Financial position
A Yes No
B No Yes
C No No
D Yes Yes
D
Explanation:
The governmental fund measurement focus is on determination of financial position and changes in financial position, rather than on net income determination. In governmental funds, the primary emphasis is on the flow of financial resources.
The following are Boa City's fixed assets: Fixed assets used in enterprise fund activities $1,000,000 Infrastructure assets 9,000,000 All other fixed assets 1,800,000What aggregate amount should Boa report in the governmental activities column of the government-wide financial statements? A $ 9,000,000 B $ 10,000,000 C $10,800,000 D $11,800,000
C
Explanation:
The government-wide financial statements (GWS) reports all general government capital assets (fixed assets plus infrastructure) in the governmental activities column ($9,000 + $1,800 = $10,800). Enterprise funds appear in the business-type activities column in the GWS.
Users need information on how the government applies restricted resources when they have a choice between using restricted or unrestricted resources to evaluate which of the following?
A
Whether the government made improper payments from the restricted and unrestricted resources
B
Whether the government complied with laws and regulations associated with federal funds
C
How ending balances may be applied to continuation of programs and services in future periods
D
None of the above
C
Explanation:
Governments rely on grants and other types of contributions to fund a variety of programs. The note disclosing how the government applies restricted resources when they have a choice between using restricted versus unrestricted resources provides users with information to evaluate how ending balances may be applied to continuation of programs and services in future periods (not whether the government made improper payments or whether the government complied with laws and regulations).
The mission of the Governmental Accounting Standards Board (GASB) includes which of the following?
A
Providing a framework to educate users and support public policies, invest-ments, and management by those users
B
Establishing standards to identify appropriate performance measures and criteria
C
Outlining best practices for administration of governmental budgets
D
Providing accounting and reporting standards for federal government agencies
A
Explanation:
The GASB’s mission includes research and development of standards that will improve accounting and financial reporting for state and local governments. The GASB’s mission includes providing a framework to educate users and support public policies, investments, and management by those users.
A county’s balances in the general fund included the following:
Appropriations $435,000 Encumbrances 18,000 Expenditures 164,000 Vouchers payable 23,000 What is the remaining amount available for use by the county?
A $230,000 B $248,000 C $253,000 D $271,000
C
Explanation:
Funds available for use = Appropriations (budgeted) - Encumbrances - Expenditures ( $435,000 - $18,000 - $164,000 ) = $253,000. Option (a) is incorrect because vouchers payable is excluded in available for funds calculations. Option (b) is incorrect because encumbrances are not excluded, instead vouchers payable are excluded from appropriations. Option (d) is incorrect because encumbrances should be excluded from appropriations to arrive at funds available.
Lys City reports a compensated absences liability in its combined balance sheet. The salary rate used to calculate the liability should normally be the rate in effect
A
When the unpaid compensated absences were earned.
B
When the compensated absences were earned or are to be paid, or at the balance sheet date, whichever results in the lowest amount.
C
At the balance sheet date.
D
When the compensated absences are to be paid.
C
Explanation:
The compensated absences liability ordinarily is measured using the pay rates in effect at the balance sheet date.
Liabilities for compensated absences should be inventoried at the end of each accounting period and adjusted to current salary costs.
Which of the following funds of a governmental unit uses the same basis of accounting as the special revenue fund?
A Internal service. B Capital projects. C Nonexpendable trust. D Enterprise.
Explanation:
The correct answer is (B)
General Funds, Special Revenue Funds, Capital Projects Fund, Debt Service Fund, and Permanent Fund use modified accrual accounting.
Under the modified accrual basis of accounting, revenue is recognized in the period it is measurable and available to spend. Costs are recorded in the period that the obligation to pay them arises, whether this is before or after the period in which the government is actually using the assets or services.
The Capital Projects Fund uses the same basis of accounting as the Special Revenue Fund.
Which basis of accounting is required for a city’s government-wide financial statements?
A Cash. B Modified Cash. C Modified Accrual. D Accrual.
Explanation:
The correct answer is (D).
A city’s government-wide financial statements are required to be completed on the accrual basis of accounting.
Which of the following is not within the scope of the GASB mission for standard setting?
A
Conceptual framework for accounting and reporting standards
B
Goals and objectives of state and local governmental services
C
External financial reporting
D
Assist users in assessing the level of services that can be provided by governmental entities
B
Explanation:
Goals and objectives for governmental programs and services should be established by management, elected officials, and the government’s stake-holders.
Which of the following is the purpose of using major fund reporting in the fund statements?
A
To highlight special revenue funds
B
To distinguish special revenue funds from debt service funds
C
To report the government’s largest funds rather than fund types
D
To provide more details about governmental funds
A
Explanation:
GASB Statement No. 34 shifted away from fund type reporting to highlight the government’s larger funds. Users do not have much interest in fund types, but they are focused on the government’s larger funds. Major fund reporting concentrates on fewer details and combines the government’s smaller funds to simplify reporting and focus on significant activities. While the determination of major funds is based on a mathematical relationship, management does have the ability to include other significant funds.
The basis of accounting for a capital projects fund is the A Cash basis. B Accrual basis. C Modified cash basis. D Modified accrual basis.
D
Explanation:
The modified accrual basis is the appropriate basis of accounting for governmental funds (i.e., General, Special Revenue, Capital Projects, Permanent, and Debt Service Funds).